Last visit was: 26 Apr 2024, 10:02 It is currently 26 Apr 2024, 10:02

Close
GMAT Club Daily Prep
Thank you for using the timer - this advanced tool can estimate your performance and suggest more practice questions. We have subscribed you to Daily Prep Questions via email.

Customized
for You

we will pick new questions that match your level based on your Timer History

Track
Your Progress

every week, we’ll send you an estimated GMAT score based on your performance

Practice
Pays

we will pick new questions that match your level based on your Timer History
Not interested in getting valuable practice questions and articles delivered to your email? No problem, unsubscribe here.
Close
Request Expert Reply
Confirm Cancel
SORT BY:
Date
Tags:
Show Tags
Hide Tags
User avatar
CEO
CEO
Joined: 15 Aug 2003
Posts: 2876
Own Kudos [?]: 1649 [25]
Given Kudos: 781
Send PM
Most Helpful Reply
Manager
Manager
Joined: 01 Feb 2018
Posts: 84
Own Kudos [?]: 209 [6]
Given Kudos: 769
Location: India
GMAT 1: 700 Q47 V38
WE:Consulting (Consulting)
Send PM
General Discussion
User avatar
Senior Manager
Senior Manager
Joined: 17 Sep 2005
Posts: 300
Own Kudos [?]: 395 [0]
Given Kudos: 0
Send PM
User avatar
Manager
Manager
Joined: 29 Oct 2008
Posts: 238
Own Kudos [?]: 1210 [2]
Given Kudos: 34
Location: United States
Concentration: Marketing, Technology
Send PM
Re: Last year company X experienced an unexpected steep drops in [#permalink]
2
Kudos
vineetgupta wrote:
Last year company X experienced an unexpected steep drops in profits.To offset this loss,company X should reduce its workforce by 10%.Doing so will allow the company to save a great deal in payroll expenditures.Company X will therefore be able to recoup the losses.

The argument above assumes that
A. The amount saved in payroll expenditures will exceed the amount lost in profits.
B. The amount saved in payroll expenditures will equal the amount lost in profits.
C. Reducing Company X's workforce will not cause the company to reduce productivity.
D. Company X has no reserve funds to offset its losses.
E. Company X has not at sometime in the past reduced its workforce.

Please give reasons for ur choices...
There are 2 inherent assumptions to Policy Proposal or GOAL/Target(FORMAT: To achieve goal, this policy shall be adhered) questions: 1) Implementation of Policy will achieve the goal. Link 2) Implementation of Policy will not impact other aspects of Goal.

It is the second of the inherent assumptions that gets reflected in Choice C.

To add to this, I feel that loss in stimulus talks about only one year of loss, whereas the conclusion(Last line: "Company X will therefore be able to recoup the losses.") and target is to recoup ie recover for losses of multiple years, and not a single year. A and B will attempt to recoup company only for this year's loss, but will not help in recouping from upcoming losses, if the employees are lost.
Manager
Manager
Joined: 28 May 2018
Posts: 58
Own Kudos [?]: 24 [1]
Given Kudos: 77
Send PM
Last year Company X experienced an unexpectedly steep drop in profits. [#permalink]
1
Kudos
Hi ,
I thought conclusion is "Company X will therefore be able to recoup the losses. " and then chose A option
Sometimes i am not able to figure out the conclusion.
Director
Director
Joined: 20 Dec 2015
Status:Learning
Posts: 876
Own Kudos [?]: 566 [1]
Given Kudos: 755
Location: India
Concentration: Operations, Marketing
GMAT 1: 670 Q48 V36
GRE 1: Q157 V157
GPA: 3.4
WE:Engineering (Manufacturing)
Send PM
Re: Last year Company X experienced an unexpectedly steep drop in profits. [#permalink]
1
Kudos
Praetorian wrote:
Last year Company X experienced an unexpectedly steep drop in profits. To offset this loss, Company X should reduce its workforce by 10%. Doing so will allow the company to save a great deal in payroll expenditures. Company X will therefore be able to recoup its losses.

The argument above assumes that


A. The amount saved in payroll expenditures will exceed the amount lost in profits.

B. The amount saved in payroll expenditures will equal the amount lost in profits.

C. Reducing Company X's workforce will not cause the company to reduce productivity.

D. Company X has no reserve funds to offset its losses.

E. Company X has not at sometime in the past reduced its workforce.


Imo C.

This is a very easy question if we see this question from real world scenario but things get muddy when the scope is only the argument. This case demands us to use real world knowledge of market and productivity. A good question to practice since it does not directly affects the conclusion.
Intern
Intern
Joined: 15 Feb 2018
Posts: 17
Own Kudos [?]: 23 [4]
Given Kudos: 32
Send PM
Re: Last year Company X experienced an unexpectedly steep drop in profits. [#permalink]
3
Kudos
If we negate option B-
The amount saved in payroll expenditures will NOT BE equal the amount lost in profits then wouldn't it break the argument? Shouldn't B correct answer?
Manager
Manager
Joined: 12 Mar 2017
Posts: 185
Own Kudos [?]: 88 [0]
Given Kudos: 87
Location: India
Concentration: Strategy, General Management
GMAT 1: 700 Q49 V37
GPA: 4
Send PM
Re: Last year Company X experienced an unexpectedly steep drop in profits. [#permalink]
shreyasawhney wrote:
If we negate option B-
The amount saved in payroll expenditures will NOT BE equal the amount lost in profits then wouldn't it break the argument? Shouldn't B correct answer?


Even I have the same question. mikemcgarry , abhimahna , VeritasKarishma . Can anybody pls enlighten?
Senior Manager
Senior Manager
Joined: 19 Oct 2013
Posts: 412
Own Kudos [?]: 307 [1]
Given Kudos: 117
Location: Kuwait
GPA: 3.2
WE:Engineering (Real Estate)
Send PM
Last year Company X experienced an unexpectedly steep drop in profits. [#permalink]
1
Kudos
shreyasawhney wrote:
If we negate option B-
The amount saved in payroll expenditures will NOT BE equal the amount lost in profits then wouldn't it break the argument? Shouldn't B correct answer?


Here is my attempt, I hope it helps.

I also faced a similar thing at the beginning but then this is what I finally understood.

When we negate option B does it break down the conclusion?

Is it necessary for the conclusion to hold true?

Negating option B, it will not equal the amount lost in profits. Well what if it doesn’t? Maybe the amount saved in payroll expenditures saves more than what is lost? Maybe the amount saved does not cover what is lost? Can we say for sure it will recoup the losses?

When we negate option C, it will reduce productivity and if it does then how can we recoup the losses? We are already losing more.

Posted from my mobile device
avatar
Intern
Intern
Joined: 27 Sep 2018
Posts: 6
Own Kudos [?]: 1 [1]
Given Kudos: 5
Send PM
Re: Last year Company X experienced an unexpectedly steep drop in profits. [#permalink]
1
Kudos
The other reason to eliminate A and B is that A, and B are clearly stated in the argument, so they are not assumption needed. Remember that assumption is the infor not stated in the argument but is needed for the author to draw the conclusion
Senior Manager
Senior Manager
Joined: 29 Jun 2020
Posts: 411
Own Kudos [?]: 477 [0]
Given Kudos: 219
Location: India
Send PM
Re: Last year Company X experienced an unexpectedly steep drop in profits. [#permalink]
Praetorian wrote:
Last year Company X experienced an unexpectedly steep drop in profits. To offset this loss, Company X should reduce its workforce by 10%. Doing so will allow the company to save a great deal in payroll expenditures. Company X will therefore be able to recoup its losses.

The argument above assumes that




Company X will reduce 10% of its workforce to offset the drop in profits. (Offset the drop ....which means the drop in profit can be covered by the reduction in workforce.)

Conclusion: Company X will recoup its losses by doing this procedure.


Predicted Answer: Reducing the workforce should not impact revenue or production.

POE :

A. The amount saved in payroll expenditures will exceed the amount lost in profits. Irrelavent, Because primise mentions that this procedure will offset the drop in profits

B. The amount saved in payroll expenditures will equal the amount lost in profits. Irrelavent, Because primise mentions that this procedure will offset the drop in profits

C. Reducing Company X's workforce will not cause the company to reduce productivity. Hmm, If prodctivity is reduced it further impacts the profit that is not accounted.

D. Company X has no reserve funds to offset its losses. Irrelavent

E. Company X has not at sometime in the past reduced its workforce.Irrelavent

IMO C.
User avatar
Non-Human User
Joined: 01 Oct 2013
Posts: 17226
Own Kudos [?]: 848 [0]
Given Kudos: 0
Send PM
Re: Last year Company X experienced an unexpectedly steep drop in profits. [#permalink]
Hello from the GMAT Club VerbalBot!

Thanks to another GMAT Club member, I have just discovered this valuable topic, yet it had no discussion for over a year. I am now bumping it up - doing my job. I think you may find it valuable (esp those replies with Kudos).

Want to see all other topics I dig out? Follow me (click follow button on profile). You will receive a summary of all topics I bump in your profile area as well as via email.
GMAT Club Bot
Re: Last year Company X experienced an unexpectedly steep drop in profits. [#permalink]
Moderators:
GMAT Club Verbal Expert
6921 posts
GMAT Club Verbal Expert
238 posts
CR Forum Moderator
832 posts

Powered by phpBB © phpBB Group | Emoji artwork provided by EmojiOne